Optimization of inscribed circle

Click For Summary
The discussion focuses on finding the maximum semi-circular area bounded by the curve y = 12 - 3x^2 and the x-axis. The zeros of the function are identified as 2 and -2, with a maximum height of 12. The radius of the inscribed semicircle is less than 2, and its center is at (0,0). The semicircle must be tangent to the parabola at the point of intersection, requiring the use of implicit differentiation to find dy/dx for both the parabola and the circle. Overall, careful consideration of the geometry and calculus is essential for solving the problem effectively.
lp27
Messages
4
Reaction score
0

Homework Statement


Given the function
y = 12- 3x^2,
find the maximum semi-circular area bounded by the curve and the x-axis.

Homework Equations



A= Pi(r^2)

The Attempt at a Solution


I found my zeros, 2 and -2, and my maximum height of 12 from the y'.

A' = 2Pi(r)
 
Last edited:
Physics news on Phys.org
Plot the function. The answer will be clear.
 
Well, no. r isn't equal to 2. If it's an inscribed semicircle bounded by the x-axis the circle must have center (0,0), agree? Draw a picture. It's a wee bit less than 2. So the equation of the circle is r^2=x^2+y^2. If it's inscribed the semicircle must be tangent to the parabola y=12-3*x^2 at the point of intersection. Find dy/dx for each and set them equal. It works out particularly easy if you find dy/dx for the circle using implicit differentiation.
 
You are, of course, correct. I should have thought about it more carefully.
 
vela said:
You are, of course, correct. I should have thought about it more carefully.

You didn't say anything wrong. I wasn't responding to your suggestion. I was responding to the original post. Plotting is always a great idea!
 
Last edited:
Question: A clock's minute hand has length 4 and its hour hand has length 3. What is the distance between the tips at the moment when it is increasing most rapidly?(Putnam Exam Question) Answer: Making assumption that both the hands moves at constant angular velocities, the answer is ## \sqrt{7} .## But don't you think this assumption is somewhat doubtful and wrong?

Similar threads

Replies
3
Views
2K
  • · Replies 4 ·
Replies
4
Views
2K
  • · Replies 7 ·
Replies
7
Views
2K
  • · Replies 2 ·
Replies
2
Views
2K
  • · Replies 14 ·
Replies
14
Views
2K
Replies
9
Views
4K
  • · Replies 8 ·
Replies
8
Views
2K
  • · Replies 3 ·
Replies
3
Views
2K
  • · Replies 1 ·
Replies
1
Views
2K
  • · Replies 1 ·
Replies
1
Views
2K